Medical ethicist: Assuming there is a reasonable chance for a cure, it is acceptable to offer experimental treatment...

texasjohnrh on June 13, 2016

Please explain

Please explain the flaw and the answer choices

Reply
Create a free account to read and take part in forum discussions.

Already have an account? log in

Mehran on July 6, 2016

@texasjohnrh the conclusion here is, "Therefore, it is never acceptable to offer experimental treatments to patients who experience no extreme symptoms of the relevant disease."

Why? Because, "Assuming there is a reasonable chance for a cure, it is acceptable to offer experimental treatments for a disease to patients who suffer from extreme symptoms of that disease. Such patients are best able to weigh a treatment's risks against the benefits of a cure."

We have a general principle here, i.e. "it is acceptable to offer experimental treatments for a disease to patients who suffer from extreme symptoms of that disease."

We can diagram this as follows:

ESD ==> AET

The contrapositive would be:

not AET ==> not ESD (i.e. if experimental treatments not acceptable, then no extreme symptoms of disease).

However, the conclusion in the stimulus was "it is never acceptable to offer experimental treatments to patients who experience no extreme symptoms of the relevant disease."

We can diagram the conclusion as follows:

not ESD ==> not AET

Notice that this is just a negation of our general principle and we know that we cannot just negate.

Stated another way, the medical ethicist is mistaking sufficient for necessary.

Now let's take a look at (C):

"Someone born and raised in a country, who has lived abroad and then returned, is exceptionally qualified to judge the merits of living in that country."

BRLAR ==> EQJM

"That is why someone who has not lived in that country should not form judgments about the merits of living there."

not BRLAR ==> not EQJM

Notice, just as we saw in the stimulus, (C) is taking the non-existence of the sufficient condition to conclude the non-existence of the necessary condition. But this is flawed logic (i.e. don't just negate!).

So (C) would be the correct answer.

Hope this helps! Please let us know if you have any other questions.